Search found 20 matches


For this problem you dont need math at all, you just need to know the properties of sets Stem: There is a five number set where the largest number is greater than the median by 3. They ask, basically whether the mean of the set is greater than the median. Remember for odd number sets that in any set...

by Zerks87

Sun Feb 06, 2011 10:59 am
Forum: Data Sufficiency
Topic: Average value > Median
Replies: 24
Views: 11240

working alone, printers x,y, and z can do a certain printing job, consisitning of a large number of pages, 12, 15, and 18 hours, respectively. What is the ratio of the time it takes printer x to do the job, working at its rate, to time it takes printers y and z to do the job, working together at th...

by Zerks87

Thu Feb 03, 2011 8:27 pm
Forum: Problem Solving
Topic: og math # 130
Replies: 132
Views: 58089

I want to take as much algebra out of this as possible. Stmt (1) sqrt 36x is an integer. The prime factors of 36 are 3 * 2 * 3 *2 (since 6*6=36). This means that both prime factors have a pair that make it a perfect square. Therefore, in order for 36x to be a perfect square, x must also be a perfect...

by Zerks87

Mon Jan 31, 2011 9:06 am
Forum: Data Sufficiency
Topic: Beat the GMAT question - Brilliant
Replies: 25
Views: 12170

If Dave works alone he will take 20 more hours to complete a task than if he worked with Diana to complete the task. If Diana works alone, she will take 5 more hours to complete the complete the task, then if she worked with Dave to complete the task? What is the ratio of the time taken by Dave to ...

by Zerks87

Sun Jan 30, 2011 9:20 am
Forum: Problem Solving
Topic: Work rate
Replies: 30
Views: 15481

... Not getting it. What does the equation look like before you get to 4x=4 or 6y=12 using the differentiation rule. I'm still stuck at the following: 2(x^2-2x)+3(y^2-4y)+18 To use the differentiation rule, you need to know differential calculus which is not a part of GMAT. You can proceed as I men...

by Zerks87

Sat Jan 29, 2011 11:57 am
Forum: Problem Solving
Topic: Minimum value of an expression
Replies: 65
Views: 40447

Hi Rahul, Can I solve it this way? Time taken to cross the man standing on the platform = 18 secs ; This would account for the time taken for the entire length of the train a point. Time taken to cross the platform = 30 secs Hence if the train had negligible length it would have taken (30-18) secs ...

by Zerks87

Wed Jan 26, 2011 9:46 am
Forum: Problem Solving
Topic: What is the length of the platform in meters...............
Replies: 46
Views: 27504

I disagree that the answer is C or E. IMO this is how to break down the question. You have a set of 13 different integers. And 30 is the median, which is defined as the middle number in the set, which means that it must be the 7th number in the set. It also says that the range of the set is 30, whic...

by Zerks87

Mon Jan 24, 2011 9:41 am
Forum: Problem Solving
Topic: Range
Replies: 31
Views: 14870

I reached answer D, please help in why it is wrong: STAT #1 x*=x ==> x(x-1)=x ==> (dividing both sides by x) (x-1) = 1 ==> x = 2 SUFFICIENT STAT #2 (x - 1 )* = (x - 2) ==> (x-1) (x-2) = (x-2) ==> (dividing both sides by (x-2)) (x-1) = 1 ==> x = 2 SUFFICIENT So why is it B and not D? thanks For stat...

by Zerks87

Fri Jan 21, 2011 11:05 am
Forum: Data Sufficiency
Topic: What is the value of x*?
Replies: 29
Views: 13522

When I did this question I did it a little differently. The question is: Is abs. x = a - b, so are both x and -x = a - b (1) x = a - b. From this we know that positive x = a - b, but -x = b - a. Therefore since one is and one isn't that means the absolute value of x cannot = a - b. Which is sufficie...

by Zerks87

Wed Jan 19, 2011 11:15 am
Forum: Data Sufficiency
Topic: Absolute DS
Replies: 34
Views: 15000

I disagree that the answer is 2500. When I look at the problem I see that there are no combinations that violate that condition that the two randomly selected number must add up to greater than 100. Therefore there are 100 numbers and 2 slots that can possibly be filled. To me this is a simple combi...

by Zerks87

Wed Jan 19, 2011 10:41 am
Forum: Problem Solving
Topic: Tokens
Replies: 27
Views: 17207

i thought A and B work together in a couple of days then A quit and B continue working on the left job? According to the question, A and B start working on the project together and A quits 10 days before the project is completed. Now mathematically the event is same as B worked alone for 10 days an...

by Zerks87

Mon Jan 10, 2011 2:40 pm
Forum: Problem Solving
Topic: A quits before the project is completed
Replies: 40
Views: 23860

So we know that both of the these are right triangles and the side relationships for a right triangle are 1, root 3, and 2 where 1 is opposite the smallest angle, root 3 is opposite the middle angle and 2 opposite the right angle. (1) angle KLM=ABC= 55 degrees. Since we know that one angle is 90, on...

by Zerks87

Mon Jan 10, 2011 2:18 pm
Forum: Data Sufficiency
Topic: whats the answer of this one and why
Replies: 37
Views: 22675

Solution Y is 30 percent liquid X and 70 percent water. If 2 kilograms of water evaporate from 8 kilograms of solutions Y and 2 kilograms of solution Y are added to the remaining 6 kilograms of liquid, what percent of this new liquid solution is liquid X? a 30% b 33 1/3% c 37 1/2% d 40% e 50% Using...

by Zerks87

Mon Jan 10, 2011 1:43 pm
Forum: Problem Solving
Topic: Hard Solutions/Mixture Problem
Replies: 24
Views: 20028

@Rahul don't you think the approach you used is longer than that above? Am only considering the 2min limit. This is a quicker way to do the question. This took me maybe a minute. The trick with this question (and I fell into it, is to forget to count 90-99 for every 100 pages) In the first 100 pgs ...

by Zerks87

Thu Dec 30, 2010 12:37 pm
Forum: Problem Solving
Topic: pages of the book
Replies: 28
Views: 16126

(1) xy > 0 implies that either both x and y should be positive or both should be negative. So, this is NOT SUFFICIENT to determine whether (-x,y) is in the same quadrant. (2) ax > 0 implies that either a and x should be positive or both should be negative. But there is no info on y. So, (2) is NOT ...

by Zerks87

Thu Dec 30, 2010 12:25 pm
Forum: Data Sufficiency
Topic: quadrant problem
Replies: 21
Views: 12402